Using a Power Series to Approximate a Definite Integral

In summary: The correct answer would be 0.000065 .You appear to be ignoring the x^2 when you're determining the coefficients for your power series.Oh... I did not keep the 4 factor all the way down. Thanks.The way to do so is of course to impose that Arctan(0) =0.
  • #1
DivGradCurl
372
0
I think I got pretty close to the answer to this problem. However, I just can't obtain the right approximation at the end. Please, help me find where I made a mistake.

Thanks.

--------------------------------------------------------------

Use a power series to approximate the definite integral to six decimal places.

[tex] \int _0 ^{1/3} x^2 \arctan \left( x^4 \right) dx [/tex]

--------------------------------------------------------------

[tex] \frac{d}{dx} \left[ \arctan \left( x^4 \right) \right] = \frac{4x^3}{1+x^8} [/tex]

[tex] \frac{1}{1+x^8}=\frac{1}{1-\left( -x^8 \right) } = \sum _{n=0} ^{\infty} \left( -x^8 \right) = \sum _{n=0} ^{\infty} \left( -1 \right) ^n x^{8n} [/tex]

[tex] \frac{d}{dx} \left[ \arctan \left( x^4 \right) \right] = 4 \sum _{n=0} ^{\infty} \left( -1 \right) ^n x^{8n+3} [/tex]

[tex] \arctan \left( x^4 \right) = 4 \sum _{n=0} ^{\infty} \int \left( -1 \right) ^n x^{8n+3} dx = \mathrm{C} + \sum _{n=0} ^{\infty} \frac{\left( -1 \right) ^n x^{8n+4}}{8n+4} [/tex]

[tex] x^2 \arctan \left( x^4 \right) dx = \mathrm{C} + \sum _{n=0} ^{\infty} \frac{\left( -1 \right) ^n x^{8n+6}}{8n+4} [/tex]

[tex] \int x^2 \arctan \left( x^4 \right) dx = \mathrm{C} + \sum _{n=0} ^{\infty} \int \frac{\left( -1 \right) ^n x^{8n+6}}{8n+4} = \mathrm{C} + \sum _{n=0} ^{\infty} \frac{\left( -1 \right) ^n x^{8n+7}}{\left( 8n+4 \right) \left( 8n+7 \right)} = \mathrm{C} + \frac{x^7}{28} - \frac{x^{15}}{180} + \frac{x^{23}}{460} - \frac{x^{31}}{868} + \cdots [/tex]

[tex] \int _0 ^{1/3} x^2 \arctan \left( x^4 \right) dx = \left[ \frac{x^7}{28} - \frac{x^{15}}{180} + \frac{x^{23}}{460} - \frac{x^{31}}{868} + \cdots \right] _0 ^{1/3} = \frac{1}{2^2\cdot 3^7\cdot 7} - \frac{1}{2^2\cdot 3^{17} \cdot 5} + \frac{1}{2^2\cdot 3^{23} \cdot 5 \cdot 23} - \frac{1}{2^2 \cdot 3^{31} \cdot 7 \cdot 31} + \cdots [/tex]

[tex] b_1 = \frac{1}{2^2\cdot 3^{17} \cdot 5} \approx 3.9 \times 10^{-10} < 10^{-6} \Longrightarrow \int _0 ^{1/3} x^2 \arctan \left( x^4 \right) dx \approx \frac{1}{2^2\cdot 3^7\cdot 7} - \frac{1}{2^2\cdot 3^{17} \cdot 5} \approx 0.000016 [/tex]

The correct answer would be [tex] 0.000065 [/tex].
 
Physics news on Phys.org
  • #2
You appear to be ignoring the [tex]x^2[/tex] when you're determining the coefficients for your power series.
 
  • #3
Oh... I did not keep the 4 factor all the way down. Thanks.
 
  • #4
thiago_j said:
I think I got pretty close to the answer to this problem. However, I just can't obtain the right approximation at the end. Please, help me find where I made a mistake.

Thanks.

--------------------------------------------------------------

Use a power series to approximate the definite integral to six decimal places.

[tex] \int _0 ^{1/3} x^2 \arctan \left( x^4 \right) dx [/tex]

--------------------------------------------------------------

[tex] \frac{d}{dx} \left[ \arctan \left( x^4 \right) \right] = \frac{4x^3}{1+x^8} [/tex]

[tex] \frac{1}{1+x^8}=\frac{1}{1-\left( -x^8 \right) } = \sum _{n=0} ^{\infty} \left( -x^8 \right) = \sum _{n=0} ^{\infty} \left( -1 \right) ^n x^{8n} [/tex]

[tex] \frac{d}{dx} \left[ \arctan \left( x^4 \right) \right] = 4 \sum _{n=0} ^{\infty} \left( -1 \right) ^n x^{8n+3} [/tex]

[tex] \arctan \left( x^4 \right) = 4 \sum _{n=0} ^{\infty} \int \left( -1 \right) ^n x^{8n+3} dx = \mathrm{C} + \sum _{n=0} ^{\infty} \frac{\left( -1 \right) ^n x^{8n+4}}{8n+4} [/tex]



[tex] x^2 \arctan \left( x^4 \right) dx = \mathrm{C} + \sum _{n=0} ^{\infty} \frac{\left( -1 \right) ^n x^{8n+6}}{8n+4} [/tex]

Why not simply use the Taylor series of Arctan? It's simply

[itex] ArcTan (x^4) = x^4 - {x^{12} \over 3} + {x^{20} \over 5} - {x^{28} \over 7} + \ldots [/itex]



Notice that you willl run into a probllem with your constant of integration "C" since it will get multiplied by [itex] x^2 [/itex] later and you would have to integrate it as well! So you must get rid of it. The way to do so is of course to impose that Arctan(0) =0.

But again, using the Taylor series of ArcTan seems simpler to me.


Pat
 

1. What is a power series?

A power series is a series of the form ∑n=0^∞ cn(x-a)n, where cn and a are constants and x is a variable. It is a mathematical tool used to approximate functions and solve differential equations.

2. How can a power series be used to approximate a definite integral?

A power series can be used to approximate a definite integral by replacing the function in the integral with its corresponding power series representation. This allows for the evaluation of the integral as a sum of infinite terms, which can be truncated to a finite number for a desired level of accuracy.

3. What are the benefits of using a power series to approximate a definite integral?

Using a power series to approximate a definite integral can provide a more accurate result compared to traditional numerical methods. It also allows for the evaluation of integrals that cannot be solved analytically.

4. What are the limitations of using a power series to approximate a definite integral?

The accuracy of the approximation depends on the convergence of the power series. If the series does not converge, the approximation will not be accurate. Additionally, the process can be time-consuming for more complex functions.

5. How can one determine the appropriate number of terms to use in a power series approximation?

The number of terms needed for a desired level of accuracy can be determined by using error estimation formulas, such as the remainder term of the Taylor series. Alternatively, the number of terms can be increased until the desired level of accuracy is achieved.

Similar threads

  • Introductory Physics Homework Help
Replies
5
Views
466
  • Introductory Physics Homework Help
Replies
2
Views
301
  • Introductory Physics Homework Help
Replies
22
Views
392
  • Introductory Physics Homework Help
Replies
10
Views
260
  • Introductory Physics Homework Help
Replies
12
Views
726
  • Introductory Physics Homework Help
Replies
3
Views
105
  • Introductory Physics Homework Help
2
Replies
64
Views
2K
  • Introductory Physics Homework Help
Replies
17
Views
386
  • Introductory Physics Homework Help
Replies
8
Views
398
  • Introductory Physics Homework Help
Replies
9
Views
586
Back
Top